Difference between revisions of "Mock AIME 3 Pre 2005 Problems/Problem 13"

m (soln?)
m (fix)
Line 7: Line 7:
  
 
==Solution==
 
==Solution==
{solution}
+
{{solution}}

Revision as of 07:54, 2 April 2007

Problem

$13.$ Let $S$ denote the value of the sum

$\left(\frac{2}{3}\right)^{2005} \cdot \sum_{k=1}^{2005} \frac{k^2}{2^k} \cdot {2005 \choose k}$

Determine the remainder obtained when $S$ is divided by $1000$.

Solution

This problem needs a solution. If you have a solution for it, please help us out by adding it.